What is the identity element in abstract algebra groups?

Click For Summary
In abstract algebra, the identity element e in groups is explored through the symmetric difference defined as A + B = (A - B) U (B - A). For e to be the identity, it must satisfy A + e = A, leading to two cases: if (A - e) = A, then e must be the empty set; if (e - A) = A, the interpretation raises questions about the definition of the identity. The discussion emphasizes that the identity cannot be undefined, as this would violate group properties. The conclusion asserts that both A and e are elements of the group, confirming that the symmetric difference must also yield valid group elements. Understanding these relationships is crucial for grasping the structure of groups in abstract algebra.
IKonquer
Messages
47
Reaction score
0
The .pdf can be ignored.

Let A + B = (A - B) U (B - A) also known as the symmetric difference.

1. Look for the identity and let e be the identity element

A + e = A
(A - e) U (e - A) = A

Now there are two cases:

1. (A - e) = A
This equation can be interpreted as removing from A all elements that belong to e to yield the set A. In order for this statement to be true, the identity element e must be the empty set.

2. (e - A) = A
This equation can be interpreted as removing from e all elements that belong to A to generate a set A. Is this statement undefined?
 

Attachments

Last edited:
Physics news on Phys.org
If A=A'(inverse) then why does A+A'={}(empty set)?
 
A + A' is the symmetric difference, and not by means of normal addition.
 
Ah. Well I learned something :)
 
(e-A) must equal something else and not A. Moreover it must equal something such that the union of (A-e)=A with (e-A)=X is A U X=A. I am sure you are aware of such a set =).

It can't be undefined or else were breaking the conditions of what it is to be a group. A and e are elements of the group so (A-e)U(e-A) must be too. right?
 
Last edited:
Question: A clock's minute hand has length 4 and its hour hand has length 3. What is the distance between the tips at the moment when it is increasing most rapidly?(Putnam Exam Question) Answer: Making assumption that both the hands moves at constant angular velocities, the answer is ## \sqrt{7} .## But don't you think this assumption is somewhat doubtful and wrong?

Similar threads

Replies
2
Views
2K
  • · Replies 7 ·
Replies
7
Views
2K
  • · Replies 26 ·
Replies
26
Views
849
  • · Replies 3 ·
Replies
3
Views
874
  • · Replies 4 ·
Replies
4
Views
3K
  • · Replies 1 ·
Replies
1
Views
2K
  • · Replies 2 ·
Replies
2
Views
2K
  • · Replies 5 ·
Replies
5
Views
2K
  • · Replies 10 ·
Replies
10
Views
2K
  • · Replies 25 ·
Replies
25
Views
11K